Đến nội dung

baopbc nội dung

Có 386 mục bởi baopbc (Tìm giới hạn từ 28-04-2020)



Sắp theo                Sắp xếp  

#609885 Cho các số từ $a_{1},...,a_{n}$...

Đã gửi bởi baopbc on 19-01-2016 - 20:58 trong Tổ hợp và rời rạc

Lời giải trong sách của Titu

Thanks anh nhiều! :like  :)




#609860 Cho các số từ $a_{1},...,a_{n}$...

Đã gửi bởi baopbc on 19-01-2016 - 20:11 trong Tổ hợp và rời rạc

Bài này hình như có trong cuốn số học của Titu Andreescu

Bài này cũng có trong cuốn các pp giải toán của Athur Angel nhưng em không dịch được, mà có dịch trên google thì cũng không hiểu gì!




#609852 Cho các số từ $a_{1},...,a_{n}$...

Đã gửi bởi baopbc on 19-01-2016 - 19:48 trong Tổ hợp và rời rạc

Cho các số từ $a_{1},...,a_{n}$ là 1 hoặc -1 thỏa mãn: S= $a_{1}a_{2}a_{3}a_{4}+a_{2}a_{3}a_{4}a_{5}+.......+a_{n}a_{1}a_{2}a_{3}=0$

Chứng minh rằng: $n\vdots 4$

P/s: Bài này có thể giải bằng bất biến! :)




#609615 Tuần 3 tháng 1/2016

Đã gửi bởi baopbc on 18-01-2016 - 18:16 trong Chuyên mục Mỗi tuần một bài toán Hình học

ta có $\frac{\overline{QI}}{\overline{BQ}}=\frac{2(\overline{BI}-\overline{BQ})}{\overline{BE}}=\frac{2BI}{BE}-1=\frac{c+a-b}{a+b+c}$

tương tự ta có $\frac{\overline{RI}}{\overline{CR}}= \frac{b+a-c}{a+b+c}$

theo định lý Ce va ta có $\frac{KC}{KB}=\frac{a+c-b}{a+b-c}$($K$ là giao của $IP$và $BC$)

$(I)$ và $BC$ tiếp xúc nhau tại $D$ ta có $K$ và $D$ đối xứng nhau qua trung điểm $BC$

lấy $L$ đối xứng $D$ qua $I$ khi đó $A,L,K$ thẳng hàng

Khi đó do $I$ trung điểm $DL$ mà $DL$ song song $AH$ nên $AP$ đi qua trung điểm $AH$

Cách giải này tương đối tự nhiên nhưng không được đẹp cho lắm, ở trên anh đã làm tắt nên nhìn khá ngắn nhưng nếu khai triển ra thì nó khá là dài. Theo em đối với những topic như thế này thì đề cao vẻ đẹp của lời giải hơn là việc đi đến lời giải./ Mong anh có thể tìm ra lời giải mới đẹp hơn./

P/s: Lời giải này chỉ hợp với thi Olympic, trong thời gian hạn hẹp thôi! :)




#609326 Tìm n nguyên dương $\geq 5$ thỏa mãn:

Đã gửi bởi baopbc on 16-01-2016 - 21:38 trong Tổ hợp và rời rạc

Tìm n nguyên dương $\geq 5$ thỏa mãn:

$i)$ 2 người bất kì quen nhau không có người quen chung.

$ii)$ 2 người bất kì không quen nhau thì có đúng hai người quen chung.

P/s: Bài này mình giải rồi nhưng không được đúng cho lắm:

Mình từng đọc một bài trong chuyên đề tổ hợp qua ánh xạ của thầy Nguyễn Chiến Thắng có một bài cấu hình tương tự nhưng chứng minh số người quen của mỗi người là như nhau, từ đó mình giải thế này: Gọi số người quen của mỗi người là x, ta có:

Nếu một người thuộc vào tập quen của người khác thì sẽ không quen với x-1 người kia( theo t/c 1)

Nếu một người cùng thuộc tập quen của hai người thì hai người đó sẽ cùng quen một người khác với người ban đầu, vậy với một người là người quen của 2 người khác thì người đó sẽ không quen $2(x-1) -1$ người. Vậy một người sẽ không quen quen tất cả là $x(x-1)-x+1=(x-1)^{2}$ vậy n có dạng $(x-1)^{2} +x+1$




#608863 [Hình học] THPT tháng 11: $UK$ đi qua điểm cố định khi $P, Q...

Đã gửi bởi baopbc on 13-01-2016 - 22:24 trong Thảo luận đề thi VMEO IV

. Mình giải bằng cách sử dụng trục đẳng phương (dĩ nhiên ngắn hơn của bạn :D )

Anh Hân đăng lời giải đi! :)




#608766 $a^2+b^2+c^2+3\geq a^2b^2+b^2c^2+c^2a^2+3a^2b^2c^2$

Đã gửi bởi baopbc on 13-01-2016 - 16:59 trong Bất đẳng thức - Cực trị

Lời giải của em thế này ạ!

Bất đẳng thức đã cho tương đương với:

$a^{2}+b^{2}+c^{2}-6+2abc(a+b+c)-3a^{2}b^{2}c^{2}\geq 0$( do $ab+bc+ac=3$)

$\Leftrightarrow (a+b+c)^{2}+2abc(a+b+c)-3a^{2}b^{2}c^{2}\geq 12$

$\Leftrightarrow (a+b+c-abc)(a+b+c+3abc)\geq 12 (*)$

Do $ab+bc+ca=3$ nên $a+b+c\geq 3,abc\leq 1$

Đặt $a+b+c=3+x$, $abc=1-y$ thì $x,y\geq 0$ và $y< 1$

$(*)\Leftrightarrow (2+x+y)(6-y+x)\geq 12$

$\Leftrightarrow x^{2}-y^{2}+8x+4y\leq 0$

Do $y< 1$ nên bất đẳng thức cuối đúng, vậy ta có đpcm./




#608456 Tuần 2 tháng 1/2016

Đã gửi bởi baopbc on 11-01-2016 - 17:11 trong Chuyên mục Mỗi tuần một bài toán Hình học

Như vậy thầy Hùng đã đưa ra lời giải bài toán Tuần 1 tháng 1 tại Tuần 2 tháng 1 kèm theo đó là bài toán mới. Xin trích dẫn lại bài toán mới:

 

Bài 21. Cho tam giác $ABC$ có tâm nội tiếp $I$ và trung tuyến $AM$. $P,Q$ lần lượt là trung điểm $IB,IC$. Lấy các điểm $K,L$ sao cho $PK \perp PA, BK \perp BA, QL \perp QA, CL \perp CA$. Chứng minh rằng $AM \perp KL$.

 

attachicon.gifScreen Shot 2016-01-11 at 10.23.09 am.png

Lời giải của em thế này:

Đường tròn đường kính $AK$ cắt đường tròn đường kính $AL$ tại $J$./

$AJ$ cắt $BC$ tại $M'$. $BC$ cắt $(AK)$ và $(AL)$ tại $E,F$.

Do $M'J$ là trục đẳng phương của $(AK)$ và $(AL)$ nên $M'B.M'E=M'C.M'F$./ Ta chứng minh $M$ trùng $M'$

$M'$ là trung điểm $BC$ khi và chỉ khi $BE=CF$./

Do $BP$ là phân giác $\angle ABC$ nên $\angle AEP=\angle PAE\Rightarrow$ tam giác $EPA$ cân.

Trên $AB$ lấy $G$ sao cho $PB=PG$ thì tam giác $PBG$ cân tại $P$ nên $\angle AGP=180o-\angle B/2=\angle EBP$

$\Rightarrow$ tam giác $AGP$= tam giác $EBP$. Vậy $BE=AG$.

Lại có:$PB=PG=PI$ nên $G$ là hình chiếu của $I$ lên $AB$.

Chứng minh tương tự: $H$ là hình chiếu của $I$ lên $AC$ nên $AH=AG$.

Vậy $BE=CF$ nên $M'$ trùng $M$

Bài toán được giải quyết xong./

P/s: Ai có thể đăng hộ em cái hình được không ạ! Em không biết đăng như thế nào?. Em xin cảm ơn./'




#608345 Tập huấn KHTN 2016

Đã gửi bởi baopbc on 10-01-2016 - 20:25 trong Hình học

Bài toán 4. Cho tam giác ABC nội tiếp đường tròn (O) với trung tuyến AM và điểm Lemoine L. AM cắt (O) tại N khác A. H là hình chiếu của N lên BC và K đối xứng H qua N. KM cắt CA,AB tại E,F. KL cắt CA,AB tại P,Q. PF cắt EQ tại R. Chứng minh rằng AL và KR cắt nhau trên (O).

Bài toán 6. Cho tam giác ABC nội tiếp đường tròn (O) và tâm nội tiếp I. Trên cạnh BC lấy các điểm E,F sao cho AB(BC+AB−AC)/BE=AC(BC+CA−AB)/CF=AB+BC+CA. Gọi M,N là trung điểm của IB,IC. Chứng minh rằng ME,NF cắt nhau trên (O).

 

 

 

Bài toán 8. Cho tam giác ABC với các trung tuyến AD,BE,CF đồng quy tại G. Các điểm X,Y,Z lần lượt thuộc EF,FD,DE sao cho nếu DX,EY,FZ đồng quy tại P thì PX=PY=PZ. Khi đó gọi Q là đẳng giác của P trong tam giác ABC. Chứng minh rằng P,Q,G thẳng hàng.

 

Bài toán 9. Cho tam giác ABC nội tiếp đường tròn (O) và trực tâm H. EF là dây cung của (O). Tia đối tia HE,HF cắt đường tròn Euler (N) của tam giác ABC tại P,Q. Gọi FP cắt EQ tại X. Chứng minh rằng bốn điểm E,F,P,Q thuộc một đường tròn (D) và XD luôn đi qua một điểm cố định khi dây EF thay đổi.

 

Bài toán 10. Cho tam giác ABC nhọn nội tiếp đường tròn (O) cố định với B,C cố định và A di chuyển trên (O). Tiếp tuyến tại A của (O) cắt BC tại T. M là trung điểm BC. Đường thẳng qua T vuông góc AM cắt CA,AB tại E,F. BE cắt CF tại D. Chứng minh rằng đường thẳng qua A vuông góc TD luôn đi qua điểm cố định khi A thay đổi.

 

Bài toán 11. Cho tam giác ABC và P là điểm bất kỳ. Gọi Oa,Ia,Ja lần lượt là tâm ngoại tiếp, tâm nội tiếp và tâm bàng tiếp đỉnh P của tam giác PBC. Tương tự có Ob,Ib,Ic,Oc,Ic,Jc. Gọi da là đường thẳng nối tâm ngoại hai tam giác PIcJb và PIbJc. Tương tự có db,dc. Chứng minh rằng da,db,dc cắt nhau tạo thành tam giác thấu xạ với tam giác OaObOc.

 

Bài toán 12. Cho tam giác ABC có đường tròn nội tiếp (I) tiếp xúc BC,CA,AB tại D,E,F. Đường thẳng qua I vuông góc IA cắt CA,AB tại A1,A2. Ab,Ac lần lượt đối xứng C,B qua A1,A2.

   

    a) Chứng minh rằng AbAc tiếp xúc (I) tại Aa.

   

    b) Tương tự có Bb,Cc. AD,BE,CF đồng quy tại Ge. Chứng minh rằng AAa,BBb,CCc đồng quy trên IGe.

 

Bài toán 13. Cho tứ giác ABCD có AB cắt CD tại E, AD cắt BC tại F và AC cắt BD tại G. Gọi AC,BD cắt EF tại P,Q. Chứng minh rằng điểm Miquel của tứ giác ABCD nằm trên đường tròn Euler của tam giác GPQ.

 

Bài toán 14. Cho tứ giác ABCD có AC cắt BD tại E. Giả sử

EA.EB.CD+EC.ED.AB=EA.ED.BC+EB.EC.AD.

Chứng minh rằng tứ giác ABCD ngoại tiếp.

 

Bài toán 15. Cho tam giác ABC nội tiếp đường tròn (O) và tâm nội tiếp I. Đường tròn (K) tiếp xúc CA,AB tại E,F và tiếp xúc trong (O) tại D. Gọi IB,IC cắt (O) tại M,N khác B,C. L là trung điểm AD. LM,LN lần lượt cắt trung trực AE,AF tại P,Q. Chứng minh rằng đường thẳng qua I vuông góc ID chia đôi PQ.

 

Bài toán 16. Cho tam giác ABC nội tiếp đường tròn (O). Đường tròn (K) tiếp xúc CA,AB tại E,F và tiếp xúc trong (O) tại D. AD cắt (K) tại L khác D. Tiếp tuyến tại L của (K) cắt (O) tại M,N. Gọi P là tâm nội tiếp tam giác DMN. Chứng minh rằng DEP+DFP=90.

 

 

Bài toán 17. Cho tam giác ABC, một đường tròn (K) đi qua B,C cắt đoạn CA,AB tại E,F. BE cắt CF tại H. P thuộc đoạn EF sao cho PAB=HAC. Gọi M,N là trung điểm BE,CF. PM,PN lần lượt cắt AB,AC tại K,L. Chứng minh rằng EF chia đôi KL.




#608341 Tập huấn KHTN 2016

Đã gửi bởi baopbc on 10-01-2016 - 20:19 trong Hình học

Bài toán 7. Cho tam giác ABC vuông tại A có đường cao AH. E,F lần lượt là hình chiếu của H lên CA,AB. Gọi K,L,N lần lượt là tâm bàng tiếp đỉnh H của các tam giác HBF,HCE,HEF. Chứng minh rằng A là tâm nội tiếp tam giác KLN.

Lời giải của mình:

Bài số 7 là hệ quả của bài toán sau:

Cho tam giác $AHB$ vuông tại $H$. $F$ là chân đường cao từ $H$ xuống $AB$.$K$ là tâm bàng tiếp góc $H$ của tam giác $BHF$.$M$ là tâm bàng tiếp góc $F$ của tam giác $BHF$. $Z$ là điểm đối xứng với $F$ qua trung điểm $BH$ thì $\angle ZMB=\angle AKF$

Áp dụng định lí Ceva sin cho tam giác $AKH$ với sự đồng quy tại $F$; cho tam giác $BMH$ với sự đồng quy tại $Z$./

Để ý rằng:$\angle FAH=\angle ZBH;\angle FHA=\angle BHZ;\angle MBZ=\angle FKH;\angle ZHM=\angle FHK$

nên:$\frac{sin\angle FKA}{sin\angle FAK}=\frac{sin\angle ZMB}{sin\angle ZMH}$

Lại có: $\angle BFH=90o$ nên $\angle KFB=\angle BMH\leq 90o$ Ta có đpcm./




#608232 $\frac{1}{a^{2}}+\frac{1...

Đã gửi bởi baopbc on 09-01-2016 - 22:07 trong Bất đẳng thức và cực trị

$\frac{1}{a^{2}}+\frac{1}{b^{2}}+\frac{1}{c^{2}}+ab+bc+ca+ab+bc+ca\geq 9\sqrt[9]{\frac{1}{a^{2}b^{2}c^{2}}.(abc)^{4}}=9\sqrt[9]{(abc)^{2}}$  mà bạn

Thành thật xin lỗi, mình bị nhầm./




#608229 $\frac{1}{a^{2}}+\frac{1...

Đã gửi bởi baopbc on 09-01-2016 - 22:01 trong Bất đẳng thức và cực trị

Cụ thể là vận dụng thế nào bạn?

$\sum \frac{1}{a^{2}}+2\sum ab\geq 9\sqrt[9]{\frac{1}{(abc)^{2}}.(abc)^{2}}=9$




#608180 Tập huấn KHTN 2016

Đã gửi bởi baopbc on 09-01-2016 - 19:50 trong Hình học

Lời giải bài số 5

:Bài toán 5. Cho tứ giác $ABCD$ có $AC$ vuông góc $BD$. Gọi $K,L,M,N$ là tâm ngoại tiếp tam giác $ABC,BCD,CDA,DAB$.

    a) Chứng minh rằng $BK,CL,DM,AN$ đồng quy tại điểm $I$

     b) Gọi $X,Y,Z,T$ là tâm ngoại tiếp tam giác $IAB,IBC,ICD,IDA$. Chứng minh rằng $X,Y,Z,T$ cùng nằm trên đường tròn $(J)$ và $XZ,YT,IJ$ đồng quy.

P/s: Bài 4 loay hoay mãi chả biết giải kiểu gì./

a, Chỉ là một ứng dụng đơn giản của hai điểm liên hợp đẳng giác. Giải băng Ceva sin là được./

b, Do $X,Y,Z,T$ là tâm ngoại tiếp các tam giác $IAB,IBC,ICA,IAD$ nên $XYZT$ là tứ giác tạo bởi đường trung trực của các cạnh $IA,IB,IC,ID$.

Bài toán quy về chứng minh $\angle DIC+\ angle AIB= 180o$./

$\angle AIB+\angle DIC=\angle CAD+\angle BDA+\angle DBC+\angle ACB=90o+90o=180o$./

Vậy $X,Y,Z,T$ đồng viên./

P/s: Hình như còn một ý thì phải./




#608168 Dựng đường tròn tiếp xúc với cả 2 đường tròn nội và ngoại tiếp tam giác

Đã gửi bởi baopbc on 09-01-2016 - 16:58 trong Hình học

Bài này có cách dựng như sau:

Thực ra trước đây có một bài toán liên quan đến cái này mà thầy em ra rồi./

ED giao OI tại S. Áp dụng định lí Menelaus ta có: SI/SO=r/R

Vậy ta xác định được S.

Cách dựng: Xác định S t/m: SI/SO=r/R

SD cắt (O) tại E. OE cắt ID tại K thì (K;KE) là w cần dựng./




#608138 Tìm GTLN $B=\dfrac{1}{\sqrt{a^2+1}...

Đã gửi bởi baopbc on 09-01-2016 - 13:23 trong Bất đẳng thức - Cực trị

Cho $a,b,c$ là các số dương thỏa mãn $6a+3b+2c=abc$. Tìm GTLN của :

$B=\dfrac{1}{\sqrt{a^2+1}}+\dfrac{2}{\sqrt{b^2+4}}+\dfrac{3}{\sqrt{c^2+9}}$

Bài này giải vậy:

Đặt a=x,b=2y,c=3z thì x+y+z=xyz./

Biểu thức đầu bài tương đương với: $\sum \frac{1}{\sqrt{x^{2}+1}}$(1)

Áp dụng bất đẳng thức AM-GM: (1)$\leq \sum \frac{1}{x+1}$

Quy đồng mẫu số kết hợp với x+y+z=xyz, ta tìm được giá trị lớn nhất là $ \frac{3}{\sqrt{3}+1}$

Dấu bằng xảy ra khi và chỉ khi x=y=z=$\sqrt{3}$




#608137 Tìm GTLN $B=\dfrac{1}{\sqrt{a^2+1}...

Đã gửi bởi baopbc on 09-01-2016 - 13:19 trong Bất đẳng thức - Cực trị

Ta có: a^2+1>=1

           b^2+4>=4

           c^2+9>=9

Bla Bla=> B<=3 => Bmax=3 <=> a=b=c=0 t/m đẳng thức 6a+3b+2c=abc 

P/s: thông cảm máy m đang bị lỗi ko đánh đc Latex

a,b,c dương mà./




#608130 Tập huấn KHTN 2016

Đã gửi bởi baopbc on 09-01-2016 - 11:11 trong Hình học

Lời giải bài số 3:Cho tứ giác hai tâm $ABCD$ có tâm ngoại tiếp $O$ và đường tròn nội tiếp $(I)$ tiếp xúc $AB,BC,CD,DA$ tại $M,N,P,Q$. $MP$ cắt $NQ$ tại $R$. $X,Y,Z,T$ là hình chiếu của $R$ lên $QM, MN, NP, PQ$. Chứng minh rằng $AX,BY,CZ,DT$ đồng quy trên $OI$

Bổ đề 1: MN,PQ,AC đồng quy./

Bổ đề 2: AC,BD,MP,NQ đồng quy./

Bổ đề 3: O,I,R thẳng hàng./

Lời giải: Gọi E là giao điểm của MN và PQ . Theo bổ đề 1 thì E thuộc AC./

Do RY //IB; RT//ID nên ta chỉ cần cm: RY/IB=RT/ID.

Theo định lý Sin, ta quy về cm:$\angle IBR= \angle RET$

Do ABCD nội tiếp, ngoại tiếp và IMBN nội tiếp, dễ dàng suy ra $\angle IBR= \angle RET$

Vậy BY,DT cắt nhau tại một điểm thuộc OI( theo bổ đề 3)

CMTT ta được q.e.d




#608126 Tập huấn KHTN 2016

Đã gửi bởi baopbc on 09-01-2016 - 10:30 trong Hình học

Bài 1 ấy công nhận là khó thật, đội không ai làm được, thầy chiều về sẽ xem kỹ cách giải bài 2 của em!

Vâng ạ, cảm ơn thầy nhiều./




#608124 Tập huấn KHTN 2016

Đã gửi bởi baopbc on 09-01-2016 - 09:59 trong Hình học

Em thử xem link trực tiếp https://www.dropbox....anQuangHung.pdf

Vâng ạ, cảm ơn thầy nhiều!

Bài toán này là bài mở đầu trong tập huấn KHTN của thầy, em không nghĩ là nó khó như vậy./

Bài số 2. Cho tam giác ABC có đường tròn nội tiếp (I) tiếp xúc BC,CA,AB tại D,E,F. Các điểm M,N thuộc (I) sao cho MN∥BC. FM,EN cắt BC tại P,Q. Chứng minh rằng đường thẳng qua P vuông góc IP, đường thẳng qua Q vuông góc IQ cắt nhau trên AD.

em giải theo kiểu trục đẳng phương thế này không biết có đúng không ạ./

Do MN // BC nên EFBC là tứ giác nội tiếp

Gọi R,S là giao của đường tròn ngoại tiếp tứ giác IPQT( T là giao của đường vuông góc với IQ tại Q với đường vuông góc với IP tại P) với (I)

Theo định lí về tâm đẳng phương thì EF,RS,PQ đồng quy( (IPQT),(EFPQ),(I) )

Gọi X là giao của TD với (I)

Ta cm: DEFX là tứ giác điều hòa tương đương với cm: DRXS là tứ giác điều hòa

K là giao của TD với (IPQT), ta có: $\angle IKD=90o$ nên IK; tiếp tuyến tại X,PQ đồng quy

Theo định lí về tâm đẳng phương cho ba đường tròn: Đường tròn đường kính ID;(IPTQ),(I) thì IK,RS,PQ đồng quy

Vậy DRXS là tứ giác điều hòa. Từ đây ta suy ra đpcm./




#608117 Tập huấn KHTN 2016

Đã gửi bởi baopbc on 09-01-2016 - 08:41 trong Hình học

Là hệ quả một bài toán đã có ở đây http://analgeomatica...n-hoc-2015.html

Không vào được trang thầy ạ!./




#608079 $\frac{1}{a^{2}}+\frac{1...

Đã gửi bởi baopbc on 08-01-2016 - 22:08 trong Bất đẳng thức và cực trị

Bài này giải vậy: cộng hai vế với 2ab+2bc+2ac thì VP=9

VT dùng AM-GM là được./




#608003 Tập huấn KHTN 2016

Đã gửi bởi baopbc on 08-01-2016 - 19:36 trong Hình học

Trong topic này. Mình sẽ đưa ra loạt bài toán trong tập huấn KHTN của thầy Trần Quang Hùng./

Bài số 1:Cho  tam giác ABC có P,Q là hai điểm liên hợp đẳng giác. K, L lần lượt là tâm nội tiếp tam giác APB và APC. KL cắt PA tại S. M là điểm tiếp xúc trong của đường tròn nội tiếp tam giác QBC với BC. CMR SL là phân giác của góc ASM./




#607876 VMO 2016: Cập nhật tình hình làm bài của các đội

Đã gửi bởi baopbc on 08-01-2016 - 09:34 trong Thi HSG Quốc gia và Quốc tế

vẫn chưa đến thời đại của 2000 ( trừ PNK )

Cho mình hỏi bạn Phạm Nam Khánh đã được đi thi quốc gia rồi cơ à./

Theo mình biết thì đâu tuyển lớp 10./

Vẫn có người sinh năm 2000 mà đi thi đó thôi

Huỳnh Bách Khoa: dogsteven

viet nam in my heart: Chuyên Vĩnh Phúc./




#607865 $\Delta AMN$ cân

Đã gửi bởi baopbc on 08-01-2016 - 08:12 trong Hình học

Câu a dễ thôi, cộng góc là được./




#607821 VMO 2016: Cập nhật tình hình làm bài của các đội

Đã gửi bởi baopbc on 07-01-2016 - 21:09 trong Thi HSG Quốc gia và Quốc tế

Có anh chị nào học PBC đi không ạ ? Em hóng :D

Có anh canhhoang30011999